MASSIMI E MINIMI SU COMPATTI (METODO DELLE CURVE DI LIVELLO)

ESERCIZIO 1

Cercare massimo e minimo assoluto di\(f\left( x,y \right)={{x}^{2}}+{{y}^{2}}-4x\) sull’insieme \(A=\left\{ \left( x,y \right)\in {{\mathbb{R}}^{2}}:\,\,1\le y\le {{e}^{x}},\,\,\,x\le 3 \right\}\) .

Le curve di livello della funzione \(f\left( x,y \right)\)sono circonferenze \({{x}^{2}}+{{y}^{2}}-4x=k\)

\({{x}^{2}}-4x+4+{{y}^{2}}=k+4\), \({{\left( x-2 \right)}^{2}}+{{y}^{2}}={{\left( \sqrt{k+4} \right)}^{2}}\), sono circonferenze di centro \(C=\left( 2,0 \right)\) e raggio \(R=\sqrt{4+k}\) .

All’aumentare di \(k\) aumenta anche il raggio della circonferenza. Pertanto bisogna individuare il punto a minima distanza da \(C\) ovvero \({{P}_{0}}\) e il punto a massima distanza cioè \({{P}_{1}}\).

In \({{P}_{0}}\) la circonferenza è tangente alla retta di equazione \(y=1\) .

\({{x}^{2}}+{{1}^{2}}-4x=k\,\,\,\Rightarrow \,\,\,{{x}^{2}}-4x+1-k\,\,=0\) . La circonferenza è tangente alla retta se \(\Delta =0\,\,\,\Rightarrow \,\,\,16-4+4k=0\,\,\,\Rightarrow \,\,\,k=-3\) . Pertanto \(\underset{x\in A}{\mathop{\min }}\,f=-3\)

Il punto a massima distanza è in \({{P}_{1}}=\left( 3,{{e}^{3}} \right)\) , punto di intersezione tra la retta e l’esponenziale.

\(\underset{x\in A}{\mathop{\max }}\,f=f\left( {{P}_{1}} \right)=9+{{e}^{6}}-12={{e}^{6}}-3\)

massimi e minimi su compatti – metodo delle curve di livello

ESERCIZIO 2

\(f\left( x,y \right)=\frac{x-y+1}{x+3}\) su \(A=\left\{ \left( x,y \right)\in {{\mathbb{R}}^{2}}:\,\,2-\frac{2}{3}{{x}^{2}}\le y\le 3-{{x}^{2}} \right\}\)

L’insieme A è rappresentato geometricamente dallo spazio compreso tra due parabole. I punti di intersezioni tra esse sono \(2-\frac{2}{3}{{x}^{2}}=3-{{x}^{2}}\,\,\,\Rightarrow \)\({{x}^{2}}=3\) \(\Rightarrow \,\,x=\pm \sqrt{3}\) , \(y=0\)

Le curve di livello di \(f\) sono un fascio di rette proprio \(\frac{x-y+1}{x+3}=k\)

Da cui: \(x-y+1=k\left( x+3 \right)\). Il centro del fascio è dato dalla soluzione del sistema:

\(\left\{ \begin{align}& x-y+1=0 \\& x+3=0 \\\end{align} \right.\,\,\,\Rightarrow \,\,\,\,C=\left( -3,2 \right)\)

Inoltre esplicitando l’equazione rispetto a \(y\) si ha che \(y=\left( 1-k \right)x+1-3k\) . Quindi il coefficiente angolare della retta è legata alla quota \(k\) tramite \(m=1-k\) . Quindi al crescere del coefficiente angolare diminuisce la quota della funzione.

metodo delle curve di livello per individuare massimi e vinimi

La quota minima si ha per in corrispondenza della retta del fascio passante per il punto \(\left( \sqrt{3},0 \right)\) . Quindi \(\underset{\left( x,y \right)\in A}{\mathop{\max }}\,f={{k}_{\min }}=\frac{\sqrt{3}+1}{\sqrt{3}+3}=\frac{\sqrt{3}+1}{\sqrt{3}\left( 1+\sqrt{3} \right)}=\frac{1}{\sqrt{3}}\)

La quota massima si in corrispondenza della retta tangente alla parabola più alta. Bisogna porre \(y=\left( 1-k \right)x+1-3k\)nell’equazione della parabola \(y=3-{{x}^{2}}\)e imporre \(\Delta =0\) .

\(\left( 1-k \right)x+1-3k=3-{{x}^{2}}\,\,\Rightarrow \,\,{{x}^{2}}+\left( 1-k \right)x-2-3k=0\,\,\,\Rightarrow \,\,\,\)

\(\Delta =1+{{k}^{2}}-2k+8+12k={{k}^{2}}+10k+9=0\,\,\Rightarrow \,\underset{\left( x,y \right)\in A}{\mathop{\min }}\,f=\,\,{{k}_{\min }}=-1\)

ESERCIZIO 3

\(f\left( x,y \right)=\log \left( x+1 \right)+\log \left( y+1 \right)\) su \(A=\left\{ \left( x,y \right)\in {{\mathbb{R}}^{2}}:\,\,{{x}^{2}}+{{y}^{2}}\le 4,x\ge 0,y\ge 0 \right\}\)

L’insieme A è un arco circonferenza delimitato dai semiassi positivi delle ascisse e delle ordinate.

\(f\left( x,y \right)=\log \left( x+1 \right)+\log \left( y+1 \right)=\log \left[ \left( x+1 \right)\left( y+1 \right) \right]\).

Il logaritmo è una funzione monotona, quindi le curve di livello della funzione \(f\left( x,y \right)\) crescono nella stessa direzione in cui crescono quelle del suo argomento, cioè se \({{c}_{2}}>{{c}_{1}}\,\,\,\Rightarrow \,\,\,\log {{c}_{2}}>\log {{c}_{1}}\) .

Cerchiamo di capire cosa rappresentano le curve di livello dell’argomento del logaritmo:

\(\left( x+1 \right)\left( y+1 \right)=c\) può essere vista come una traslazione del luogo geometrico definito in maniera implicita dall’equazione \(xy=c\), di un vettore \(v=\left( -1,-1 \right)\) .

\(xy=c\) è una iperbole avente come asintoti gli assi cartesiani e posizionata nel primo e terzo quadrante se \(c>0\), nel secondo e quarto quadrante \(c<0\). Se invece \(c=0\) l’iperbole degenera in due rette (asse x e asse y).

In particolare siamo interessati al caso \(c>0\)

metodo delle curve di livello per individuare massimi e minimi

Al crescere di \(c\) le curve di livello si spostano verso l’alto, e il vertice delle iperboli si sposta lungo la bisettrice del primo e terzo quadrante. Il massimo della funzione sull’insieme \(A\) (tenendo presente l’andamento delle curve di livello in figura e le simmetrie del problema), si ha sulla circonferenza in coordinate \({{P}_{\max }}=\left( \sqrt{2},\,\,\sqrt{2} \right)\) e vale \(f\left( {{P}_{\max }} \right)=\log \left( 5+\sqrt{2} \right)\) , mentre il minimo si trova nell’origine e vale \(f\left( 0,0 \right)=0\)

MASSIMI E MINIMI SU COMPATTI (PARAMETRIZZAZIONE DEL BORDO)

ESERCIZIO 4

Data la funzione \(f\left( x,y \right)=y={{x}^{2}}+{{y}^{2}}-\frac{x}{2}\) cercare massimo e minimo assoluti della funzione su \(C=\left\{ \left( x,y \right)\in {{\mathbb{R}}^{2}}:\,\,{{x}^{2}}+{{y}^{2}}\le 1,\,\,\,{{\left( x-1 \right)}^{2}}+{{\left( y+1 \right)}^{2}}\le 1 \right\}\)

L’insieme \(C\) è costituito dall’interno di una circonferenza centrata nell’origine e di raggio unitario e l’esterno di una circonferenza di raggio unitario e centrata in \(c=\left( 1,-1 \right)\) .

Cerchiamo i punti stazionari: \(\nabla f=0\) :

\(\left\{ \begin{align}& 2x-\frac{1}{2}=0 \\& 2y=0 \\\end{align} \right.\,\,\Rightarrow \,\,\,{{P}_{0}}=\left( \frac{1}{4},0 \right)\,\in \,\,C\,\,\,\Rightarrow \,\,\,f\left( {{P}_{0}} \right)=\frac{1}{16}-\frac{1}{8}=-\frac{1}{16}\)

parametrizzazione del bordo e teorema di Weierstress per individuare massimi e minimi

Il bordo dell’insieme può essere parametrizzato con due archi di circonferenza \({{\gamma }_{1}}\) e \({{\gamma }_{2}}\).

Tra i potenziali massimi e minimi bisogna prendere in considerazione anche l’intersezione tra \({{\gamma }_{1}}\) e \({{\gamma }_{2}}\) in \({{P}_{1}}=\left( 0,-1 \right)\) e \({{P}_{2}}=\left( 1,0 \right)\).

Il bordo del primo arco di circonferenza può essere parametrizzato con \({{\gamma }_{1}}=\left( \cos t,\sin t \right)\), dove \(t\in \left[ 0,\frac{3}{2}\pi \right]\).

\({{\phi }_{1}}\left( t \right)=f\left( \cos t,\sin t \right)={{\cos }^{2}}t+{{\sin }^{2}}t-\frac{\cos t}{2}=1-\frac{\cos t}{2}\)

\({{\phi }_{1}}^{\prime }\left( t \right)=\frac{\sin t}{2}=0\) \(\Rightarrow \) \(t=\left\{ 0,\pi \right\}\)

Per \(t=0\) si ha \({{P}_{2}}=\left( \cos 0,\sin 0 \right)=\left( 1,0 \right)\) , \(f\left( {{P}_{2}} \right)=\phi \left( 0 \right)=\frac{1}{2}\)

Per \(t=\pi \) si ha \({{P}_{3}}=\left( \cos \pi ,\sin \pi \right)=\left( -1,0 \right)\), \(f\left( {{P}_{3}} \right)=\phi \left( \pi \right)=\frac{3}{2}\)

\(f\left( {{P}_{4}} \right)=\phi \left( \frac{3}{2}\pi \right)=1\)

Il bordo del secondo arco di circonferenza può essere parametrizzato traslando la curva \({{\gamma }_{0}}=\left( \cos t,\sin t \right)\,\,\,t\in \left( \frac{\pi }{2},\pi \right)\) di un vettore\(c=\left( 1,-1 \right)\). Quindi si ottiene la parametrizzazione \({{\gamma }_{2}}={{\gamma }_{0}}+c=\left( \cos t+1,\sin t-1 \right)\,\,\,\,\,t\in \left( \frac{\pi }{2},\pi \right)\) .

\({{\phi }_{1}}\left( t \right)=f\left( \cos t+1,\sin t-1 \right)={{\left( \cos t+1 \right)}^{2}}+{{\left( \sin t-1 \right)}^{2}}-\frac{\cos t+1}{2}=\)

\({{\cos }^{2}}t+2\cos t+1+{{\sin }^{2}}t-2\sin t+1-\frac{1}{2}\cos t-\frac{1}{2}=3+\frac{3}{2}\cos t-2\sin t\)

Derivando rispetto a \(t\) si ha:

\({{\phi }_{1}}^{\prime }\left( t \right)=-\frac{3}{2}\sin t-2\cos t=0\,\,\,\Rightarrow \,\,\,\frac{\sin t}{\cos t}=-\frac{4}{3}\,\,\,\,\Rightarrow \,\,\tan t=-\frac{4}{3}\)

La tangente è negativa nel secondo e quarto quadrante. Noi siamo interessati in particolare alla soluzione nel secondo quadrante, perché \(t\in \left( \frac{\pi }{2},\pi \right)\) .

Il vettore \(\left( \cos t,\sin t \right)\) , si ottiene normalizzando il vettore \(v=\left( -3,4 \right)\), quindi \(\left( \cos t,\sin t \right)=\frac{v}{\left\| v \right\|}=\left( -\frac{3}{5},\frac{4}{5} \right)\) .

Senza calcolare esplicitamente \(t\) , è possibile ricavare le coordinate del punto sostituendo il valore di seno e coseno in \({{\gamma }_{2}}=\left( \cos t+1,\sin t-1 \right)\). Quindi \({{P}_{4}}=\left( -\frac{3}{5}+1,\,\,\,\frac{4}{5}-1 \right)=\left( \frac{2}{5},-\frac{1}{5} \right)\)

Da cui \(f\left( {{P}_{4}} \right)=\frac{4}{5}\) .

Il minimo e massimo assoluti sono:

\(\underset{\left( x,y \right)\in \,\,C}{\mathop{\min }}\,\,f\left( x,y \right)=-\frac{1}{16}\,\,\,,\,\,\,\underset{\left( x,y \right)\in \,\,C}{\mathop{\max }}\,\,f\left( x,y \right)=\frac{3}{2}\,\,\)

ESERCIZIO 5

Si vuole individuare punti di massimo e minimo della funzione\(f\left( x,y \right)={{x}^{2}}y\) , lungo il vincolo definito in maniera implicita dalla parabola di equazione: \(g\left( x,y \right)={{x}^{2}}+2x-y-2=0\)

SOLUZIONE 1 (MOLTIPLICATORI DI LAGRANGE)

\(\left\{ \begin{align}& 2xy=\lambda \left( 2x+2 \right) \\& {{x}^{2}}=-\lambda \\& {{x}^{2}}+2x-y-2=0 \\\end{align} \right.\)

Il vincolo è una parabola, non è un insieme limitato e quindi non vale il teorema di Weierstress.

\(\left\{ \begin{align}& xy=-{{x}^{2}}\left( x+1 \right) \\& \lambda =-{{x}^{2}} \\& {{x}^{2}}+2x-y-2=0 \\\end{align} \right.\)

Soluzione 1:

\(\left\{ \begin{align}& x=0 \\ & \lambda =0 \\ & {{x}^{2}}+2x-y-2=0 \\\end{align} \right.\)

\(\left\{ \begin{align}& x=0 \\  & \lambda =0 \\  & y=-2 \\ \end{align} \right.\)

\({{P}_{0}}=\left( 0,-2 \right)\)

Soluzione 2:

\(\left\{ \begin{align}  & y=-x\left( x+1 \right) \\  & \lambda =-{{x}^{2}} \\  & {{x}^{2}}+2x-y-2=0 \\ \end{align} \right.\)

\(\left\{ \begin{align}  & y=-x\left( x+1 \right) \\  & \lambda =-{{x}^{2}} \\  & {{x}^{2}}+2x+x\left( x+1 \right)-2=0 \\ \end{align} \right.\)

\(\left\{ \begin{align}  & y=-x\left( x+1 \right) \\  & \lambda =-{{x}^{2}} \\  & 2{{x}^{2}}+3x-2=0 \\ \end{align} \right.\)

Il secondo sistema da a sua volta due soluzioni:

\(\left\{ \begin{align}  & y=-\frac{3}{4} \\  & \lambda =-\frac{1}{4} \\  & x=\frac{1}{2} \\ \end{align} \right.\) \(\vee \) \(\left\{ \begin{align}  & y=-2 \\  & \lambda =-4 \\  & x=-2 \\ \end{align} \right.\) \({{P}_{1}}=\left( -2,-2,-4 \right),{{P}_{2}}=\left( \frac{1}{2},-\frac{3}{4}, \right)\)

\(\underset{\left( x,y \right)\to \left( \pm \infty ,+\infty \right)}{\mathop{\lim }}\,f\left( x,y \right)=+\infty \) , \(f\left( 0,-2 \right)=0,\,\,\,f\left( -2,-2 \right)=-8,\,\,\,f\left( \frac{1}{2},-\frac{3}{4} \right)=-\frac{3}{16}\)

Le frecce nel grafico mostrano la direzione in cui la funzione \(f\left( x,y \right)\) percorrendo la parabola risulta crescente.

Si ha che \({{P}_{1}}\) è minimo assoluto, \({{P}_{0}}\)è massimo relativo e \({{P}_{2}}\) è minimo relativo. Massimo assoluto non esiste perché la funzione va ad infinito percorrendo la parabola verso l’alto.

massimi e minimi vincolati metodo della parametrizzazione della curva

SOLUZIONE 2: PARAMETRIZZAZIONE DELLA PARABOLA

\(f\left( x,y \right)={{x}^{2}}y\)

\(g\left( x,y \right)={{x}^{2}}+2x-y-2=0\)

\(y={{x}^{2}}+2x-2\)

\(\phi \left( x \right)=f\left( x,{{x}^{2}}+2x-2 \right)={{x}^{2}}\left( {{x}^{2}}+2x-2 \right)={{x}^{4}}+2{{x}^{3}}-2{{x}^{2}}\)

\({\phi }’\left( x \right)=4{{x}^{3}}+6{{x}^{2}}-4x=2x\left( 2{{x}^{2}}+3x-2 \right)=0,\,\,\,\Rightarrow \,\,{{x}_{0}}=-2\,\,,\,\,\,{{x}_{1}}=0,\,\,\,{{x}_{2}}=\frac{1}{2}\)

\(\underset{x\to \pm \infty }{\mathop{\lim }}\,\phi \left( x \right)=+\infty \),\(\phi \left( -2 \right)=-8\), \(\phi \left( 0 \right)=0\),\(\phi \left( \frac{1}{2} \right)=-\frac{3}{16}\)

ESERCIZIO 6

Individuare massimi e minimi assoluti di\(f\left( x,y \right)={{e}^{{{x}^{2}}+2xy-{{y}^{2}}}}\) sul vincolo \(C=\left\{ \left( x,y \right)\in {{\mathbb{R}}^{2}}:\frac{{{x}^{2}}}{4}+\frac{{{y}^{2}}}{6}=1 \right\}\)

SOLUZIONE

Poiché l’esponenziale è una funzione monotona su tutto l’asse dei reali allora cercare massimi e minimi di \(f\left( x,y \right)\) è lo stesso che cercare massimi e minimi di \({{f}_{1}}\left( x,y \right)={{x}^{2}}+2xy-{{y}^{2}}\)

La Lagrangiana della funzione\({{f}_{1}}\left( x,y \right)\) sull’insieme \(C\) diventa \(\Lambda \left( x,y,\lambda \right)={{x}^{2}}+2xy-{{y}^{2}}-\lambda \left( \frac{{{x}^{2}}}{4}+\frac{{{y}^{2}}}{6}-1 \right)\) .

Ponendo \(\nabla \Lambda =0\) si ottiene il sistema:

\(\left\{ \begin{align} & 2x+2y=\lambda \frac{x}{2} \\  & 2x-2y=\lambda \frac{y}{3} \\ & \frac{{{x}^{2}}}{4}+\frac{{{y}^{2}}}{6}=1 \\ \end{align} \right.\)

\(\left\{ \begin{align}& \left( 4-\lambda  \right)x+4y=0 \\  & 6x+\left( -\lambda -6 \right)y=0 \\ & 3{{x}^{2}}+2{{y}^{2}}=12 \\ \end{align} \right.\)

Le prime due equazioni costituiscono un sistema lineare omogeneo \(\mathbf{Ax}=\mathbf{0}\) , dove \(\mathbf{A}\) è data da:

\(\mathbf{A}=\left[ \begin{matrix}4-\lambda & 4 \\6 & -\lambda -6 \\\end{matrix} \right]\)

Il sistema ammette soluzione se \(\det \mathbf{A}=0\) , oppure \(x=y=0\) . La seconda soluzione è da scartare perché non appartiene al vincolo \(C\).

\(\det \mathbf{A}={{\lambda }^{2}}+2\lambda -48=0\,\,\,\,\Rightarrow \,\,\,\,{{\lambda }_{1,2}}=\left\{ -8,6 \right\}\) . Da cui:

\(\left\{ \begin{align} & \lambda =6 \\  & -2x+4y=0 \\  & 3{{x}^{2}}+2{{y}^{2}}=12 \\ \end{align} \right.\)

\(\left\{ \begin{align}  & \lambda =6 \\  & x=2y \\  & 14{{y}^{2}}=12 \\ \end{align} \right.\)

\({{P}_{0}}=\left( 2\sqrt{\frac{6}{7}},\sqrt{\frac{6}{7}} \right),{{P}_{1}}=\left( -2\sqrt{\frac{6}{7}},-\sqrt{\frac{6}{7}} \right)f\left( {{P}_{0}} \right)=f\left( {{P}_{1}} \right)={{e}^{6}}\)

\(\left\{ \begin{align}  & \lambda =-8 \\  & 12x+4y=0 \\  & 3{{x}^{2}}+2{{y}^{2}}=12 \\ \end{align} \right.\)

\(\left\{ \begin{align}  & \lambda =-8 \\  & y=-3x \\  & 21{{x}^{2}}=12 \\ \end{align} \right.\)

\({{P}_{2}}=\left( \frac{2}{\sqrt{7}},-\frac{6}{\sqrt{7}} \right),{{P}_{3}}=\left( -\frac{2}{\sqrt{7}},\frac{6}{\sqrt{7}} \right)f\left( {{P}_{2}} \right)=f\left( {{P}_{3}} \right)={{e}^{-8}}\)

L’insieme \(C\)essendo rappresentato da un ellissi è chiuso e limitato e la funzione e continua su di esso e per Weierstress massimo e minimo esistono sicuramente e valgono \(\underset{C}{\mathop{\max }}\,f\left( x,y \right)={{e}^{6}}\) e \(\underset{C}{\mathop{\min }}\,f\left( x,y \right)={{e}^{-8}}\).

Le curve di livello della funzione \(f\left( x,y \right)\) si ottengono ponendo costante l’esponente e sono delle iperboli.